Autor Tema: Area de superficie 1

0 Usuarios y 1 Visitante están viendo este tema.

24 Octubre, 2020, 03:56 am
Leído 874 veces

weimar

  • $$\Large \color{#5372a0}\pi\,\pi$$
  • Mensajes: 97
  • País: br
  • Karma: +0/-0
Hallar el area de la esfera \( x^2+y^2+z^2=12 \) que no se encuentra en el interior del paraboloide \( z=x^2+y^2 \)
Hola, parametrize la esfera \(  \varphi(\theta,\phi )=(2\sqrt{3}\cos \theta \sin \phi,2\sqrt{3}\sin \theta \sin \phi,2\sqrt{3}\cos \phi) \)
com \( \theta \in[0,2\pi] \) e \( \phi  \)  varia en que intervalo??? :banghead: :banghead:

24 Octubre, 2020, 04:49 am
Respuesta #1

Richard R Richard

  • Ingeniero Industrial
  • $$\Large \color{#5b61b3}\pi\,\pi\,\pi\,\pi\,\pi$$
  • Mensajes: 2,319
  • País: ar
  • Karma: +1/-0
  • Sexo: Masculino
  • Dentro de la ciencia todo,fuera de la ciencia nada

si \( z=x^2+y^2 \)



\( x^2+y^2+z^2=12  \)


la esfera intersecta al paraboloide en


\( z+z^2=12 \)


resuelves la cuadrática, tomas el valor positivo ya que el negativo no se condice con la primer ecuación  resultando \( z=3 \)


cuando \( y=0 \) si \( z=3 \) entonces \( x=\sqrt 3 \)  y cuando \( x=0 \)  resulta \( y=\sqrt 3 \)


luego \( \phi =arctan\dfrac{z}{x} =arctan\dfrac{z}{y}= arctan\sqrt 3=\dfrac{\pi}{3} \)


así los limites de \( \phi \) seran  \( -\dfrac{\pi}{2} \) y \( \dfrac{\pi}{3} \)
Saludos  \(\mathbb {R}^3\)

24 Octubre, 2020, 01:39 pm
Respuesta #2

weimar

  • $$\Large \color{#5372a0}\pi\,\pi$$
  • Mensajes: 97
  • País: br
  • Karma: +0/-0
Hola, porque el \( \phi \) inicia  en el  \( -\pi/2 \).  :-\ :-\ :-\
En general yo uso que \( \phi \in [0,\pi] \) , entonces siendo asi no seria por el enunciado que: \( \phi \in [\pi/3,\pi] \) :-\

24 Octubre, 2020, 06:37 pm
Respuesta #3

Richard R Richard

  • Ingeniero Industrial
  • $$\Large \color{#5b61b3}\pi\,\pi\,\pi\,\pi\,\pi$$
  • Mensajes: 2,319
  • País: ar
  • Karma: +1/-0
  • Sexo: Masculino
  • Dentro de la ciencia todo,fuera de la ciencia nada

Como parametrizas el tercer y cuarto cuadrante? es decir las z negativas, para mi eso son los ángulos de \(  \phi  \) entre \( -\pi/2 \) y \( 0 \), luego sigues subiendo hasta que\(  \phi=\pi/3 \)




el paraboloide solo intersecta la esfera en las z positivas , en la parte negativa tienes la semiesfera entera.


Saludos  \(\mathbb {R}^3\)

24 Octubre, 2020, 07:21 pm
Respuesta #4

weimar

  • $$\Large \color{#5372a0}\pi\,\pi$$
  • Mensajes: 97
  • País: br
  • Karma: +0/-0
Hola , sustituyendo en la integral para calcular el area,  usado  $$\phi \in [-\pi/2,\pi/3]$$ sale

$$A=\int_{0}^{2\pi}\int_{-\pi/2}^{\pi/3} 12 \sin \phi  d \phi   d \theta=-12\pi  \sqrt{3}  $$

Ahora usando mi $$\phi \in [\pi/3, \pi]$$ sale

$$A=\int_{0}^{2\pi}\int_{\pi/3}^{\pi} 12 \sin \phi  d \phi   d \theta=12\pi (\sqrt{3}+2)  $$ que si resulta igual a la respuesta que colocaron aqui. Que estraño  :-\

24 Octubre, 2020, 11:31 pm
Respuesta #5

Luis Fuentes

  • el_manco
  • Administrador
  • Mensajes: 55,996
  • País: es
  • Karma: +0/-0
Hola

Hola , sustituyendo en la integral para calcular el area,  usado  $$\phi \in [-\pi/2,\pi/3]$$ sale

$$A=\int_{0}^{2\pi}\int_{-\pi/2}^{\pi/3} 12 \sin \phi  d \phi   d \theta=-12\pi  \sqrt{3}  $$

Ahora usando mi $$\phi \in [\pi/3, \pi]$$ sale

$$A=\int_{0}^{2\pi}\int_{\pi/3}^{\pi} 12 \sin \phi  d \phi   d \theta=12\pi (\sqrt{3}+2)  $$ que si resulta igual a la respuesta que colocaron aqui. Que estraño  :-\

Es que tu tomas esféricas tomando como ángulo cero el polo Norte y ángulo \( \pi \) el Sur:

\( \varphi(\theta,\phi )=(2\sqrt{3}\cos \theta \color{red}\sin \phi \color{black},2\sqrt{3}\sin \theta \color{red} \sin \phi\color{black},2\sqrt{3} \color{red}\cos \phi\color{black}) \)

 Entonces los límites correctos son $$\phi \in [\pi/3, \pi]$$.

Richard estaba pensando en esta otra parametrización:

\( \varphi(\theta,\phi )=(2\sqrt{3}\cos \theta \color{red}\cos \phi \color{black},2\sqrt{3}\sin \theta \color{red} \cos \phi\color{black},2\sqrt{3} \color{red}\sin \phi\color{black}) \)

Saludos.

25 Octubre, 2020, 12:14 am
Respuesta #6

delmar

  • Moderador Global
  • Mensajes: 3,548
  • País: pe
  • Karma: +0/-0
  • Sexo: Masculino
Hola

Solamente para complementar, el parámetro \( \phi \) que utilizar Richard R Richard es el ángulo entre el radio vector del punto y el plano XY, considerando negativos los que están debajo del plano y positivos los que están encima, en esas condiciones la parametrización es la que ha puesto Luis Fuentes y ahí \( \phi \in{[\frac{-\pi}{2},\frac{\pi}{3}]} \) en esas condiciones el integrando es \( 12 cos \phi \), verifica.

El parámetro  \( \phi \) que utilizas es el ángulo entre el radio vector del punto y el semieje positivo z en ese caso la parametrización es la que menciona Luis Fuentes, evidentemente hay un pequeño error tipográfico \( \phi\in{[\frac{\pi}{6}, \pi]} \), es decir el integral que has puesto esta mal (los límites) pero sale igual, verifica el integral correcto es :

\( \int_{0}^{2 \pi}\int_{\pi/6}^{\pi} 12 sen \phi \ d \phi d \theta \)




Saludos

25 Octubre, 2020, 01:02 am
Respuesta #7

Richard R Richard

  • Ingeniero Industrial
  • $$\Large \color{#5b61b3}\pi\,\pi\,\pi\,\pi\,\pi$$
  • Mensajes: 2,319
  • País: ar
  • Karma: +1/-0
  • Sexo: Masculino
  • Dentro de la ciencia todo,fuera de la ciencia nada


Richard estaba pensando en esta otra parametrización:

\( \varphi(\theta,\phi )=(2\sqrt{3}\cos \theta \color{red}\cos \phi \color{black},2\sqrt{3}\sin \theta \color{red} \cos \phi\color{black},2\sqrt{3} \color{red}\sin \phi\color{black}) \)

Saludos.





Si  ahora veo que no se corresponde a la forma en que él la planteaba....  con esa  se parte del complemento de \( \pi /3 \)   que es \( \pi/6 \) hasta \( \pi \)
Saludos  \(\mathbb {R}^3\)

25 Octubre, 2020, 01:08 am
Respuesta #8

weimar

  • $$\Large \color{#5372a0}\pi\,\pi$$
  • Mensajes: 97
  • País: br
  • Karma: +0/-0
Gracias muchados,  muy buena la aclaracion que  $$\phi \in [\pi/6,\pi]$$  entendi ahora.